26

Friend asked me this question and I didn't manage to solve it with basic thermodynamic reasoning. I believe this problem is easly solvable via numeric methods by choosing specific systems, though I prefer an analytic, more general and more intuitive solution.

Two different and isolated systems (which specified by $S_1(E_1,V_1,N_1)$ and $S_2(E_2,V_2,N_2)$) are seperatly prepeard to satisfy particular $(P,T)$ requirements, so that $P_1=P_2=P$ but $T_1 \ne T_2$. Afterwards the two systems are brough one near the other, with a single piston (unmovable at first) seperating them. The piston doesn't allow transfer of heat or particles at any stage. After the two systems were properly juxtaposed the restriction on the movement of the piston is removed. Will the piston move from its original position?

One way of treatment suggests that since $P_1=P_2$ and and since only mechanichal work (exchange) is allowed - the piston will not move.

Other way sugest that by forcing maximum entropy (thermodynamic equilibrium) for the combined system, we will get $dS_{tot}=dS_1+dS_2=0$, and in particular (since there is only one degree of freedom here) $\frac{\partial S_1}{\partial V_1}=\frac{\partial S_2}{\partial V_1}$ so at equilibrium $\frac{P_1}{T_1}=\frac{P_2}{T_2}$, hence the piston will move.

Alexander
  • 2,324
  • 2
    The two treatments aren't contradictory. If you only let the systems exchange mechanical work (i.e. the piston is a perfect insulator) then they'll never come to thermodynamic equilibrium. – knzhou May 25 '16 at 01:06
  • 2
    But the question is actually pretty tricky. It's indeterminate how much the piston will move; thermodynamics alone can't tell you. – knzhou May 25 '16 at 01:08
  • 1
    If you consider the force balance on the released piston,the piston can be in force equilibrium only if the two pressures are equal. The only way that can happen is if the piston does not move. – Chet Miller May 25 '16 at 01:25
  • the answer is the first, not because of thermodynamics but because of newton's laws. I cannot follow the logic of the second argument though. –  May 25 '16 at 02:05
  • 2
    look at the link posted in accepted answer: http://physics.stackexchange.com/questions/105344/system-in-mechanical-but-not-thermal-equilibrium – dan-ros May 27 '16 at 15:29
  • An interesting question related to this: can you actually bring the two systems together without changing the entropy of the system away from $S_1+S_2$. That moment where you bring them together is not an equilibrium state, so it can be hard to define entropy "correctly." Entropy is most easily defined in thermodynamic equilibrium. – Cort Ammon May 27 '16 at 16:30
  • @tonydo: yes, this question looks like an exact duplicate of that one. (But cannot be closed due to the open bounty.) – L. Levrel May 30 '16 at 14:59
  • @ChetMiller that is true for a piston of finite mass. But we can reformulate the problem using a mass-less piston. Then the piston velocity can change in time even while the pressures are equal. – Ján Lalinský Dec 30 '23 at 15:27
  • @JánLalinský Are you talking about Browning collision effects? Otherwise, how can the piston move if the forces on the two faces of the piston cancel (even if it is massless)? – Chet Miller Dec 30 '23 at 16:05
  • @ChetMiller no I mean macroscopic theory. Bodies can move without forces, this is the core of Newton's 1st law. Accelerating a mass-less body requires zero force, so it is allowed. – Ján Lalinský Dec 30 '23 at 16:09
  • But the gas itself has mass. If the two pressures are equal, how does the piston know which direction to go? – Chet Miller Dec 30 '23 at 16:21
  • @ChetMiller We have to allow it in analysis, even if the reason for symmetry breaking is not clear from the current state. Why is Earth moving in one direction and not the opposite one? We tend to seek cause of motion in the past state. But whatever the cause is, we have to allow for motion when net force is zero. This is similar to how heat flow between two bodies of equal temperature is allowed. Re gas itself has mass, motion of the piston implies there is net force on the gas. – Ján Lalinský Dec 30 '23 at 16:26

6 Answers6

15

You've discovered a famous problem in thermodynamics.

In our case the piston will not move. The mechanical argument is right, while the maximum entropy argument is inconclusive.

To see that $P_1=P_2$ is an equilibrium position you can also apply conservation of energy. Since there is no heat exchange,

$$dU_{1,2} = -P_{1,2} dV_{1,2}$$

We require that $dU=0$ since our system is isolated from the environment, hence

$$dU_1 + dU_2 = 0 \to P_1 d V_1 + P_2 dV_2 = 0$$

But $V=V_1+V_2$ and $V$ is fixed, so that $dV_1 = - dV_2$ and we obtain

$$P_1=P_2$$

Now let's see the entropy maximum principle. The problem is that you forgot that $S$ is a function of energy too:

$$S(U,V)= S_1 (U_1, V_1)+ S_2 (U_2, V_2)$$

$$d S = dS_1 + dS_2 = \frac{dU_1}{T_1} + \frac{P_1}{T_1} dV_1 + \frac{dU_2}{T_2} + \frac{P_2}{T_2} dV_2$$

Since $dU_{1,2} = -P_{1,2} dV_{1,2}$, we see that $dS$ vanishes identically, so that we can say nothing about $P_{1,2}$ and $T_{1,2}$: the entropy maximum principle is thus inconclusive.

Update

Your question actually inspired me a lot of thoughts in the past days and I found out that...I was wrong. I basically followed the argument given by Callen in his book Thermodynamics (Appendix C), but it looks like:

  1. There are some issues with the argument itself
  2. I misinterpreted the argument

My error was really silly: I only showed that $P_1=P_2$ is a necessary condition for equilibrium, not that it is a sufficient condition, i.e. (if the argument is correct and) if the system is at equilibrium, then $P_1=P_2$, but if $P_1=P_2$ the system could still be out of equilibrium...which it is!

I am still not really able to explain why the whole argument is wrong: some authors have said that equilibrium considerations should follow from the second law and not from the first and that the second law is not inconclusive. You can read for example this article and this article. They both use only thermodynamics considerations, but I warn you that the second tries to contradict the first. So the problem, from a purely thermodynamic point of view, is really difficult to solve without making mistakes, and I have found no argument that persuaded me completely and for good.

This article takes into consideration exactly your problem and shows that the piston will move, making the additional assumption that the gases are ideal gases.

We take the initial temperatures, T1 and T2, to be different, and the initial pressures, p1 and p2, to be equal. Once unblocked, the piston gains a translational energy to the right of order 1/2KT1 from a collision with a side 1 molecule, and a translational energy to the left of order 1/2KT2 from a collision with a side 2 molecule . In this way energy passes mainly from side 2 to side 1 if T2>T1.

[...] In this process just considered, the pressures on the two sides of the piston are equal at all times, which means no "work" is done. However, the energy transfer occurs through the agency of the moving piston, and if one considers "work" to be the energy transferred via macroscopic, non-random motion, then it appears that "work" is done.

This is really similar to the argument given by Feynman in his lectures (39-4). Feynman basically uses kinetic theory arguments to show that if we start with $P_1 \neq P_2$ the piston will at first "slosh back and forth" (cit.) until $P_1 = P_2$, and then, due to random pressure fluctuations, slowly converge towards thermodynamic equilibrium ($T_1=T_2$).

The argument is really tricky because we assume that if the pressure is the same on both side the piston will not move, forgetting that pressure is just $2/3$ of the density times the average kinetic energy per particle

$$P = \frac 2 3 \rho \langle \epsilon_K \rangle$$

just like temperature is basically the average kinetic energy (without the density multiplicative factor). So we are dealing with statistical quantities, which are not "constant" from a microscopic point of view. So while thermodynamically we say that if $P_1=P_2$ the piston won't move, from a microscopic point of view it will actually slightly jiggle back and forth because of the different collision it experiences from particles in the left and right sides.

There have been also simulations of your problem which show that if we start with $P_1=P_2$ and $T_1\neq T_2$ the piston will oscillate until we reach thermodynamic equilibrium ($T_1=T_2$). See the pictures below, which I took from the article.

enter image description here

enter image description here

valerio
  • 16,231
  • Your original answer was OK. It's easy to see that $P₁=P₂$ is necessary and sufficient for $U₁=U₂$ given that $V=\text{const.}$ and $Q=0$ (you can reverse the order of your demonstration, it's still valid). Indeed $S=\text{const.}$ because work doesn't change entropy. As far as thermodynamics stricto sensu (that is, not statistical mechs) is concerned, you're right and the piston doesn't move. – L. Levrel May 30 '16 at 15:06
  • Now, neither 1st nor 2nd principle can tell if a state is a state of equilibrium, because thermodynamics deals with equilibrium states only. Given two states A and B, 1st principle tells if B can be reached from A or A from B (namely: if they have the same $U$). 2nd principle tells which of A→B or B→A is possible (that which increases $S$). As @MartinKochanski told, this doesn't even say if this possible change will happen. The system will spontaneously evolve to maximum entropy when constraints are removed. As long as the piston is adiabatic there can be no change in $S$ as you showed. – L. Levrel May 30 '16 at 15:13
  • Also, in the article quote you give, there is a clear shortcoming: with equal pressure and different temperatures, there must be different densities, so the rate of collisions is different on both sides and you cannot directly conclude that energy flows either way. – L. Levrel May 30 '16 at 15:19
  • I am aware of the fact that I can formally reverse the steps and show that $P_1=P_2$ implies $dU=dU_1+dU_2=0$ ($U_1=U_2$ is incorrect. Think about the case of an ideal gas: its energy is a function of temperature only so if $T_1\neq T_2$ then $U_1\neq U_2$), but this tells us only that energy is conserved and nothing about equilibrium. Callen himself writes in his book, after having obtained $P_1=P_2$ from conservation of energy: “this necessary but not sufficient characterization of the equilibrium state is in agreement with our expectation on physical grounds.” (Thermodynamics, appendix C) – valerio May 31 '16 at 10:00
  • Also, you say that “neither 1st nor 2nd principle can tell if a state is a state of equilibrium”, but this is false. The closure relation $dS=0$ is the condition for equilibrium. Actually, it is the definition of equilibrium, as it follows from the 2nd law: thermodynamical equilibrium is when entropy is at a maximum, so its variation must vanish. Finally, would you please specify what article are you referring to, since I quoted many? – valerio May 31 '16 at 10:01
  • Sorry for the miswriting, I meant $U_\text{final}=U_\text{initial}$ of course (it would be silly to conclude about an extensive quantity with only intensive information, even $T_1=T_2$ is not enough since we know neither $n_{1/2}$ nor $V_{1/2}$); shouldn't be posting physics in the late evening :-S 2) As I emphasized, a system evolves to the state of maximum entropy with respect to some degree of freedom once constraints on this DoF are released. Please read the full comment to understand what I mean (a short sentence can never render subtleties). 3) You cited many but quoted only 1 :-)
  • – L. Levrel May 31 '16 at 19:00
  • @L.Levrel Ok, sorry, thought cite and quote had the same meaning. Now that I read the article more throughly, I don't find it so convincing (this guy claims to solve the problem using only thermodynamics, but imposes equal final temperature on both sides). It looks like that everyone trying to solve the problem using only thermodynamics treats it in a completely different way. As a matter of fact, I asked a question myself to understand what is wrong with Callen's argument... – valerio Jun 01 '16 at 18:39
  • @L.Levrel Regarding the 2nd law, I think I understand what you are saying, but I also think that what many of the authors (and alsoMartin Kochanski) are trying to say is that thermodynamics cannot find a conclusive answer (it can only find that $P_1=P_2$ at equilibrium but not that $T_1=T_2$) because a thing like an "adiabatic movable piston" does not exist, it is an absurdum. To be really adiabatic the piston should have infinite mass, but in that case the problem would be trivial. – valerio Jun 01 '16 at 18:44
  • For me, many concepts of thermodynamics do not exist, as they are ideal/limiting behaviors: reversibility, quasi-static-ity (quasi-stasis?), additivity... Yet, the theory as a whole is consistent. Here, for me, the fact that $dS$ vanishes tells that the system is in neutral equilibrium. Now, this is all formal, because this is an ideal/limiting case. But if we were to build a real system approaching this ideal one, we would find the mechanical equilibrium to be much faster than the thermal one; so the ideal result is quite good. – L. Levrel Jun 02 '16 at 13:30
  • @valerio excuse me, your final answer is that the piston WILL move until it reaches both P1 = P2 and T1 = T2, right? I'm asking because your first answer and your later edit say the opposite from each other – Juan Perez Jul 31 '22 at 16:11